Solving Dieterici's Equation: Critical Pressure for a,b & R

  • Thread starter Thread starter John O' Meara
  • Start date Start date
Click For Summary
Dieterici's equation relates pressure, volume, and temperature of a gas using constants a, b, and R. The critical volume is defined as v_c = 2b and the critical temperature as T_c = a/(4bR). The discussion focuses on verifying the first and second derivatives of pressure with respect to volume at critical conditions, ultimately leading to the determination of critical pressure p_c. Participants explore the differentiation process and the implications of treating temperature as a constant while solving the equations. The conversation emphasizes the complexity of deriving the second derivative and the interdependence of variables in the equations.
John O' Meara
Messages
325
Reaction score
0
Dieterici's equation ( an alternative to Van der waal's equation) states that the pressure p, volume v and absolute temperature T of a mass of gas are connected by the equation
p=\frac{RT}{(v-b)}\exp{\frac{-a}{vRT}} \\,
where a, b and R are constants. Verify that both \frac{{\partial p}}{{\partial v}} = 0 \\
and \frac{{\partial^2 p}}{{\partial^2 v}} =0 \\, for the critical volume and temperature v_c and T_c respectively,where v_c = 2band T_c = \frac{a}{4bR} \\. What is the value of p_c the critical pressure in terms of a,b and e?
I have a question in solving this: namely is \frac{d\exp{\frac{-a}{vRT}}}{dv} = \frac{d \exp{\frac{-a}{vRT}}}{d v^{-1}} \frac{d v^{-1}}{dv} \\ Because I don't think so: could someone explain what the l.h.s. is equal to. Thanks for the help.
 
Last edited:
Physics news on Phys.org
\frac{d exp(\frac{-\alpha}{vRT})}{dv}= exp(\frac{-\alpha}{vRT})\frac{d\frac{-\alpha}{vRT}}{dv}= exp(\frac{-\alpha}{vRT})\frac{\alpha}{v^2RT}

That's because
\frac{d \frac{A}{v}}{dv}= \frac{d Av^{-1}}{dv}= -Av^{-2}
for any constant A.
 
HallsofIvy said:
\frac{d exp(\frac{-\alpha}{vRT})}{dv}= exp(\frac{-\alpha}{vRT})\frac{d\frac{-\alpha}{vRT}}{dv}= exp(\frac{-\alpha}{vRT})\frac{\alpha}{v^2RT}

Ok, I finally got to this step. But how do I take the 2nd derivative of this last result? It's gnarly.
 
Ok,

I think I got the 2nd derivative, and then I set both 1st and 2nd derivative to zero.
Now I have 3 equations (original, 1st derivative, 2nd derivative), but how many unknowns? I know that V is an unknown, but isn't T also an unknown? I treated it as a constant.

Do I solve for V in the 1st derivative and plug it into the 2nd derivative?

This is all very confusing . . .
 
\frac{{\partial p}}{{\partial v}} = \frac{{\partial}}{{\partial v}} (\frac{RT}{v-b}\exp^{\frac{-a}{vRT}}) \\
which = \exp^{\frac{-a}{vRT}} \frac{{\partial }}{{\partial v}}(\frac{RT}{v-b}) + \frac{RT}{v-b} \frac{{\partial }}{{ \partial v}}(\exp^{\frac{-a}{vRT}}) \\.
Now use HallsofIvy's equation to evaluate the second term of the product rule expression to get the following:
\frac{RT}{(v-b)^2}\exp^{\frac{-a}{vRT}} - \frac{a}{(v-b)v^2} \exp^{\frac{-a}{vRT}} \\ = \exp^{\frac{-a}{vRT}}(\frac{RT}{(v-b)^2} - \frac{a}{(v-b)v^2}) \\ \mbox{ For the critical volume } \ v_c \ \mbox{ and the critical temperature } \ T_c \\ \ \frac{{\partial p}}{{\partial v}}= \exp^{-2}( \frac{a}{4b(b)^2} - \frac{a}{b4b^2})=0
 
Question: A clock's minute hand has length 4 and its hour hand has length 3. What is the distance between the tips at the moment when it is increasing most rapidly?(Putnam Exam Question) Answer: Making assumption that both the hands moves at constant angular velocities, the answer is ## \sqrt{7} .## But don't you think this assumption is somewhat doubtful and wrong?

Similar threads

  • · Replies 3 ·
Replies
3
Views
5K
Replies
2
Views
1K
Replies
4
Views
1K
  • · Replies 12 ·
Replies
12
Views
2K
  • · Replies 18 ·
Replies
18
Views
3K
  • · Replies 3 ·
Replies
3
Views
2K
Replies
30
Views
4K
Replies
7
Views
2K
Replies
5
Views
2K
  • · Replies 1 ·
Replies
1
Views
1K